Álgebra Lineal II: Unicidad de la forma canónica de Jordan

Por Leonardo Ignacio Martínez Sandoval

Introducción

En la entrada anterior enunciamos el teorema de la forma canónica de Jordan y demostramos la existencia de dicha forma bajo ciertas hipótesis. Como corolario, quedó pensar cuál es la versión para matrices. En esta entrada enunciamos la versión para matrices (totalmente equivalente a la de transformaciones lineales) y nos enfocamos en mostrar la unicidad de la forma canónica de Jordan.

Unicidad de la forma canónica de Jordan

El siguiente teorema es totalmente análogo al enunciado en la entrada anterior. Recuerda que $\leq$ es un orden total fijo de $F$ (en $\mathbb{R}$, es el orden usual).

Teorema. Sea $A$ una matriz $M_n(F)$ cuyo polinomio característico $\chi_A(X)$ se divide en $F$. Entonces, existen únicos valores $\lambda_1\leq \ldots \leq \lambda_n$ en $F$ y únicos enteros $k_1,\ldots,k_d$ tales que \begin{align*} &k_1+k_2+\ldots+k_d = n,\\ &k_1\leq k_2 \leq \ldots \leq k_d,\end{align*} para los cuales $A$ es similar a la siguiente matriz de bloques de Jordan:

$$\begin{pmatrix} J_{\lambda_1,k_1} & 0 & \cdots & 0 \\ 0 & J_{\lambda_2,k_2} & \cdots & 0 \\ \vdots & & \ddots & \vdots \\ 0 & 0 & \cdots & J_{\lambda_d,k_d}\end{pmatrix}.$$

Usaremos esta versión para demostrar la unicidad, lo cual también implicará la unicidad para la versión de transformaciones lineales.

Mediante la demostración de existencia de la entrada anterior, llegamos a que si el polinomio característico de $A$ es

$$\chi_A(X)=(X-\lambda_1)^{m_1}(X-\lambda_2)^{m_2}\cdots(X-\lambda_r)^{m_r},$$

entonces $A$ es similar a una matriz conformada por matrices de bloques de Jordan $J_1,J_2,\ldots,J_r$, en donde cada $J_i$ es de tamaño $m_i$ y de bloques de Jordan de eigenvalor $\lambda_i$.

Si $A$ fuera similar a otra matriz $K$ de bloques de Jordan, podríamos agrupar por eigenvalores de los bloques $\kappa_1< \ldots < \kappa_s$ en matrices de bloques de Jordan tamaños $o_1,\ldots,o_s$, digamos $K_1,\ldots,K_s$. El polinomio característico de $K$ sería entonces

$$\chi_{K}(X)=(X-\kappa_1)^{o_1}(X-\kappa_2)^{o_2}\cdots(X-\kappa_s)^{o_s}.$$

Pero $K$ es similar a $A$, y entonces deben tener el mismo polinomio característico, así que conciden en raíces y multiplicidad. Esto demuestra que $r=s$ y como los $\lambda_i$ y los $\kappa_i$ están ordenados, también demuestra las igualdades $\lambda_i=\kappa_i$ y $m_i=o_i$ para todo $i\in\{1,\ldots,r\}.$

Sólo nos queda argumentar la igualdad entre cada $J_i$ y $K_i$ para $i\in\{1,\ldots,r\}$. Pero ambas una forma canónica de Jordan para la transformación nilpotente que se obtiene de restringir $T_{A-\lambda_i I}$ a $\ker(T_{A-\lambda_i I}^{m_i})$. Por la unicidad que demostramos para la forma canónica de Jordan para transformaciones nilpotentes, concluimos que $J_i=K_i$. Esto termina la demostración de la unicidad de la forma canónica de Jordan.

$\square$

Una receta para encontrar la forma canónica de Jordan

Ya con el teorema demostrado, ¿cómo juntamos todas las ideas para encontrar la forma canónica de Jordan de una matriz $A$ en $M_n(F)$ cuyo polinomio característico se divida en $F$? Podemos proceder como sigue.

  1. Encontramos el polinomio característico $\chi_A(X)$ y su factorización, digamos $$\chi_A(X)=(X-\lambda_1)^{m_1}(X-\lambda_2)^{m_2}\cdots(X-\lambda_r)^{m_r}.$$
  2. Nos enfocamos en encontrar las matrices de bloque de Jordan $J_i$ para cada eigenvalor $\lambda_i$. Sabemos que la matriz $J_i$ será de tamaño $m_i$.
  3. Para saber exactamente cuál matriz de bloques de Jordan es $J_i$, pensaremos en que tiene $b_1,b_2,\ldots,b_{m_i}$ bloques de Jordan de eigenvalor $\lambda_i$ de tamaños $1,2, \ldots,m_i$. Consideramos la matriz $A_i=A-\lambda_i I$. Los $b_1,\ldots,b_{m_i}$ son la solución al siguiente sistema de ecuaciones en las variables $x_1,\ldots,x_{m_i}$.
    \begin{align*}
    m_i&= 1\cdot x_1 + 2\cdot x_2 + 3 \cdot x_3 + \ldots + m_i \cdot x_{m_i}\\
    m_i-n+\text{rango}(A_i-\lambda_i I)&=0\cdot x_1 + 1\cdot x_2 + 2 \cdot x_3 + \ldots + (m_i-1) \cdot x_{m_i}\\
    m_i-n+\text{rango}({A_i-\lambda_i I}^2)&= 0 \cdot x_1 + 0 \cdot x_2 + 1 \cdot x_3 + \ldots + (m_i-2)\cdot x_{m_i}\\
    m_i-n+\text{rango}({A_i-\lambda_i I}^3)&= 0 \cdot x_1 + 0 \cdot x_2 + 0 \cdot x_3 + \ldots + (m_i-3)\cdot x_{m_i}\\
    &\vdots\\
    m_i-n+\text{rango}({A_i-\lambda_i I}^{m_i-1})&= 0\cdot x_1 + 0 \cdot x_2 + 0 \cdot x_3 + \ldots + 1 \cdot x_{m_i}.
    \end{align*}
  4. Juntamos todos los $J_i$ en una misma matriz y los ordenamos apropiadamente.

El paso número $3$ está motivado por lo que sabemos de las matrices nilpotentes, y es bueno que pienses por qué se estudia específicamente ese sistema de ecuaciones para cada eigenvalor $\lambda_i$ y multiplicidad $m_i$.

Ejemplo de obtener la forma canónica de Jordan

Veamos un ejemplo del procedimiento descrito en la sección anterior.

Ejemplo. Encontraremos la forma canónica de Jordan de la siguiente matriz: $$A=\begin{pmatrix}-226 & -10 & -246 & 39 & 246\\234 & 23 & 236 & -46 & -236\\-198 & -20 & -192 & 41 & 195\\-93 & 10 & -122 & 10 & 122\\-385 & -30 & -393 & 74 & 396\end{pmatrix}.$$

Con herramientas computacionales, podemos darnos cuenta de que el polinomio característico de esta matriz es $$\chi_A(X)=X^{5} – 11 X^{4} + 46 X^{3} – 90 X^{2} + 81 X- 27.$$

Este polinomio se puede factorizar como $$(X-1)^2(X-3)^3.$$ Así, la submatriz de bloques de Jordan $J_1$ de eigenvalor $1$ tendrá tamaño $2$ y la $J_3$ de eigenvalor $3$ tendrá tamaño $3$. Pero, ¿de qué tamaño son cada uno de los bloques de Jordan en cada una de estas matrices?

Para respondernos esto para $J_1$, notamos que sus bloques son de tamaño $1$ y $2$ solamente. Si hay $b_1$ bloques de tamaño $1$ y $b_2$ bloques de tamaño $2$, por la teoría desarrollada arriba tendremos:

\begin{align*}
b_1+2b_2&=2\\
b_2&=2-5+\text{rango}(A-I)=2-5+4=1.
\end{align*}

El rango de $A-I$ lo obtuvimos computacionalmente, pero recuerda que también puede ser obtenido con reducción gaussiana. Resolviendo el sistema, $b_2=1$ y entonces $b_1=0$. Concluimos que en $J_1$ hay un bloque de Jordan de tamaño $2$.

Para $J_3$, reciclemos las variables $b_i$ (para no introducir nuevas). Los bloques pueden ser de tamaño $1,2,3$. Supongamos que de estos tamaños respectivamente hay $b_1,b_2,b_3$ bloques. Los $b_i$ cumplen:

\begin{align*}
b_1+2b_2+3b_3&=3\\
b_2+2b_3&=3-5+\text{rango}(A-3I)=3-5+3=1\\
b_3&=3-5+\text{rango}((A-3I)^2)=3-5+2=0.
\end{align*}

Así, $b_3=0$, y en consecuencia $b_2=1$ y entonces $b_1=1$. Concluimos que $J_3$ tiene un bloque de tamaño $1$ y uno de tamaño $3$. Por lo tanto, la forma canónica de Jordan de $A$ es:

$$\begin{pmatrix} J_1 & 0 \\ 0 & J_3 \end{pmatrix} = \begin{pmatrix} J_{1,2} & 0 & 0 \\ 0 & J_{3,1} & 0 \\ 0 & 0 & J_{3,2} \end{pmatrix} = \begin{pmatrix}1 & 1 & 0 & 0 & 0\\0 & 1 & 0 & 0 & 0\\0 & 0 & 3 & 0 & 0\\0 & 0 & 0 & 3 & 1\\0 & 0 & 0 & 0 & 3\end{pmatrix}$$

$\triangle$

Otro problema sobre forma canónica de Jordan

La receta anterior funciona en general y da la forma canónica de Jordan. Esto es algo que probablemente en la práctica en aplicaciones no tendrás que hacer manualmente nunca, pues hay herramientas computacionales que te pueden ayudar. Sin embargo, es importante entender con profundidad el teorema y la receta de manera teórica, pues hay problemas conceptuales en los que no podrás usar herramientas computacionales. A continuación veremos un ejemplo.

Problema. Sea $A$ una matriz en $M_6(\mathbb{R})$ con polinomio característico $$\chi_A(X)=X^6-2X^4+X^2.$$

  • ¿Cuántas posibilidades hay para la forma canónica de Jordan de $A$?
  • Demuestra que si el rango de $A$ es $5$, entonces $A$ no es diagonalizable.

Solución. Podemos factorizar el polinomio característico de $A$ como sigue:

$$\chi_A(X)=X^2(X+1)^2(X-1)^2.$$

Así, la forma canónica de Jordan está conformada por una matriz de bloques de Jordan $J_0$ de eigenvalor $0$ y tamaño $2$; una $J_1$ de eigenvalor $1$ y tamaño $2$; y una $J_{-1}$ de eigenvalor $-1$ y tamaño $2$.

Cada $J_i$ tiene dos chances: o es un bloque de Jordan de tamaño $2$, o son dos bloques de Jordan de tamaño $1$. Así, en total tenemos $2\cdot 2 \cdot 2=8$ posibilidades.

Si $A$ es de rango $5$, entonces tendríamos en las cuentas de cantidad de bloques $b_1$ y $b_2$ para eigenvalor $0$ que

\begin{align*}
b_1+2b_2&=2\\
b_2&=2-6+\text{rango}(A)=2-6+5=1,
\end{align*}

de donde en $J_0$ tendría $1$ bloque de tamaño $2$ y ninguno de tamaño $1$. Si $A$ fuera diagonalizable, su diagonalización sería una forma canónica de Jordan donde para eigenvalor $0$ se tendrían $2$ bloques de tamaño $1$ y ninguno de tamaño $2$. Así, $A$ tendría dos formas canónicas de Jordan distintas, lo cual es imposible.

$\square$

Más adelante…

Con esta entrada terminamos de demostrar el teorema de la forma canónica de Jordan, uno de los teoremas más bonitos de álgebra lineal. ¿Te das cuenta de todo lo que utilizamos en su demostración? Forma matricial de transformaciones lineales, el teorema de Cayley-Hamilton, polinomio característico, subespacios estables, teoría de dualidad, sistemas de ecuaciones lineales, resultados auxiliares de polinomios, etc. Es un resultado verdaderamente integrador.

En la siguiente entrada, la última del curso, hablaremos de algunas de las consecuencias del teorema de la forma canónica de Jordan. Discutiremos cómo lo podemos utilizar para clasificar a las matrices por similaridad. Veremos una aplicación con respecto a una matriz y su transpuesta. También, esbozaremos un poco de por qué en cierto sentido el resultado no sólo vale para las matrices cuyo polinomio se divide sobre el campo, sino que para cualquier matriz. Con ello terminaremos el curso.

Tarea moral

  1. Calcula la forma canónica de Jordan $J$ de la matriz $$A=\begin{pmatrix} 1 & 0 & -3 \\ 1 & -1 & -6 \\ -1 & 2 & 5 \end{pmatrix}.$$ Además de encontrar $J$, encuentra de manera explícita una matriz invertible $P$ tal que $A=P^{-1}JP$.
  2. Calcula la forma canónica de Jordan de la matriz $$\begin{pmatrix} 1 & 1 & 0 & 0 \\ 0 & 1 & 2 & 0 \\ 0 & 0 & 1 & 0 \\ 0 & 0 & 0 & 2 \end{pmatrix}$$
  3. Explica y demuestra cómo obtener lo siguiente para una matriz de bloques de Jordan:
    • Su polinomio característico.
    • Su polinomio mínimo.
    • Su determinante.
    • Su traza.
    • Sus eigenespacios.
  4. Justifica con más detalle por qué la receta que se propone para calcular la forma canónica de Jordan en efecto funciona. Necesitarás varios de los argumentos que dimos en la entrada anterior.
  5. Demuestra que una matriz $A\in M_n(F)$ para la cual su polinomio característico se divide en $F$ es diagonalizable si y sólo si cada bloque de cada matriz de bloques de la forma canónica de Jordan tiene tamaño $1$.

Entradas relacionadas

Agradecimientos

Trabajo realizado con el apoyo del Programa UNAM-DGAPA-PAPIME PE109323 «Hacia una modalidad a distancia de la Licenciatura en Matemáticas de la FC-UNAM – Etapa 3»

Deja una respuesta

Tu dirección de correo electrónico no será publicada. Los campos obligatorios están marcados con *

Este sitio usa Akismet para reducir el spam. Aprende cómo se procesan los datos de tus comentarios.